[Rozgrzewka OM][MIX][Nierówności] Nierówności

Zadania z kółek matematycznych lub obozów przygotowujących do OM. Problemy z minionych olimpiad i konkursów matematycznych.
Regulamin forum
Wszystkie tematy znajdujące się w tym dziale powinny być tagowane tj. posiadać przedrostek postaci [Nierówności], [Planimetria], itp.. Temat może posiadać wiele różnych tagów. Nazwa tematu nie może składać się z samych tagów.
Awatar użytkownika
Premislav
Użytkownik
Użytkownik
Posty: 15687
Rejestracja: 17 sie 2012, o 13:12
Płeć: Mężczyzna
Lokalizacja: Warszawa
Podziękował: 196 razy
Pomógł: 5220 razy

Re: [Rozgrzewka OM][MIX][Nierówności] Nierówności

Post autor: Premislav »

Proszę znaleźć największą taką stałą \(\displaystyle{ A}\), że nierówność
\(\displaystyle{ \left(x_{1}^{3}+x_{2}^{3}+x_{3}^{3}+1\right)\left(y_{1}^{3}+y_{2}^{3}+y_{3}^{3}+1\right)\left(z_{1}^{3}+z_{2}^{3}+z_{3}^{3}+1\right)\ge A(x_{1}+y_{1}+z_{1})(x_{2}+y_{2}+z_{2})(x_{3}+y_{3}+z_{3}) }\)
zachodzi dla dowolnych liczb rzeczywistych dodatnich \(\displaystyle{ x_{i}, \ y_{i}, \ z_{i}, \ i\in\left\{1,2,3\right\}}\)
i dla tej stałej \(\displaystyle{ A}\) proszę znaleźć wszystkie wartości \(\displaystyle{ x_{i}, \ y_{i}, \ z_{i}}\), dla których zachodzi równość.
Tmkk
Użytkownik
Użytkownik
Posty: 1718
Rejestracja: 15 wrz 2010, o 15:36
Płeć: Mężczyzna
Lokalizacja: Ostrołęka
Podziękował: 59 razy
Pomógł: 501 razy

Re: [Rozgrzewka OM][MIX][Nierówności] Nierówności

Post autor: Tmkk »

Mam nadzieję, że nic nie pokręciłem.

Ukryta treść:    
Awatar użytkownika
Premislav
Użytkownik
Użytkownik
Posty: 15687
Rejestracja: 17 sie 2012, o 13:12
Płeć: Mężczyzna
Lokalizacja: Warszawa
Podziękował: 196 razy
Pomógł: 5220 razy

Re: [Rozgrzewka OM][MIX][Nierówności] Nierówności

Post autor: Premislav »

Świetnie, możesz kontynuować.
Tmkk
Użytkownik
Użytkownik
Posty: 1718
Rejestracja: 15 wrz 2010, o 15:36
Płeć: Mężczyzna
Lokalizacja: Ostrołęka
Podziękował: 59 razy
Pomógł: 501 razy

Re: [Rozgrzewka OM][MIX][Nierówności] Nierówności

Post autor: Tmkk »

Dla rzeczywistych dodatnich, z warunkiem \(\displaystyle{ a+b+c \ge 3}\) pokazać, że zachodzi

\(\displaystyle{ \frac{1}{a^2+b+c}+\frac{1}{a+b^2+c}+\frac{1}{a+b+c^2} \le 1}\)
Awatar użytkownika
Premislav
Użytkownik
Użytkownik
Posty: 15687
Rejestracja: 17 sie 2012, o 13:12
Płeć: Mężczyzna
Lokalizacja: Warszawa
Podziękował: 196 razy
Pomógł: 5220 razy

Re: [Rozgrzewka OM][MIX][Nierówności] Nierówności

Post autor: Premislav »

Ukryta treść:    
Tmkk
Użytkownik
Użytkownik
Posty: 1718
Rejestracja: 15 wrz 2010, o 15:36
Płeć: Mężczyzna
Lokalizacja: Ostrołęka
Podziękował: 59 razy
Pomógł: 501 razy

Re: [Rozgrzewka OM][MIX][Nierówności] Nierówności

Post autor: Tmkk »

Tak, bardzo fajnie. Twoja kolej.
Awatar użytkownika
Premislav
Użytkownik
Użytkownik
Posty: 15687
Rejestracja: 17 sie 2012, o 13:12
Płeć: Mężczyzna
Lokalizacja: Warszawa
Podziękował: 196 razy
Pomógł: 5220 razy

Re: [Rozgrzewka OM][MIX][Nierówności] Nierówności

Post autor: Premislav »

Dla liczb dodatnich \(\displaystyle{ a,b,c}\) spełniających zależność \(\displaystyle{ abc=1}\) proszę wykazać, że
\(\displaystyle{ \frac{a}{(a+1)(b+1)}+\frac{b}{(b+1)(c+1)}+\frac{c}{(c+1)(a+1)}\ge \frac{3}{4} }\)
i rozstrzygnąć, kiedy zachodzi równość w nierówności.
Awatar użytkownika
arek1357
Użytkownik
Użytkownik
Posty: 5742
Rejestracja: 6 gru 2006, o 09:18
Płeć: Mężczyzna
Lokalizacja: blisko
Podziękował: 130 razy
Pomógł: 525 razy

Re: [Rozgrzewka OM][MIX][Nierówności] Nierówności

Post autor: arek1357 »

Z góry przepraszam, że się wtrącam i zakłócam spokój ponieważ nie powinno mnie tu być w tej sekcie ale po sprowadzeniu do wspólnego mianownika mamy:

\(\displaystyle{ \frac{ab+ac+bc+a+b+c}{(a+1)(b+1)(c+1)} \ge \frac{3}{4} }\)

po wymnożeniu dołów mamy:

\(\displaystyle{ \frac{ab+ac+bc+a+b+c}{ab+ac+bc+abc+a+b+c+1} \ge \frac{3}{4} , abc=1 }\)

po wymnożeniu i skróceniu mamy:

\(\displaystyle{ ab+ac+bc+a+b+c \ge 6}\) , ?

\(\displaystyle{ ab+ac+bc+a+b+c \ge 3 \sqrt[3]{(abc)^2} +3 \sqrt[3]{abc} =3+3=6}\)

a=b=c=1, równość...

cnd...
Ostatnio zmieniony 17 maja 2020, o 22:45 przez arek1357, łącznie zmieniany 1 raz.
Awatar użytkownika
Premislav
Użytkownik
Użytkownik
Posty: 15687
Rejestracja: 17 sie 2012, o 13:12
Płeć: Mężczyzna
Lokalizacja: Warszawa
Podziękował: 196 razy
Pomógł: 5220 razy

Re: [Rozgrzewka OM][MIX][Nierówności] Nierówności

Post autor: Premislav »

Jest dobrze, możesz wrzucać następne zadanie.
Awatar użytkownika
arek1357
Użytkownik
Użytkownik
Posty: 5742
Rejestracja: 6 gru 2006, o 09:18
Płeć: Mężczyzna
Lokalizacja: blisko
Podziękował: 130 razy
Pomógł: 525 razy

Re: [Rozgrzewka OM][MIX][Nierówności] Nierówności

Post autor: arek1357 »

Sorki ale pauzuję daję Tobie...wyprowadzam się..., jestem tu persona non grata..
Awatar użytkownika
Premislav
Użytkownik
Użytkownik
Posty: 15687
Rejestracja: 17 sie 2012, o 13:12
Płeć: Mężczyzna
Lokalizacja: Warszawa
Podziękował: 196 razy
Pomógł: 5220 razy

Re: [Rozgrzewka OM][MIX][Nierówności] Nierówności

Post autor: Premislav »

Dla liczb rzeczywistych dodatnich spełniających warunek \(\displaystyle{ a_{1}a_{2}\ldots a_{n}=1}\) proszę udowodnić nierówność
\(\displaystyle{ \sqrt{1+a_{1}^{2}}+\sqrt{1+a_{2}^{2}}+\ldots+\sqrt{1+a_{n}^{2}}\le \sqrt{2}(a_{1}+a_{2}+\ldots+a_{n}) }\)
bosa_Nike
Użytkownik
Użytkownik
Posty: 1664
Rejestracja: 16 cze 2006, o 15:40
Płeć: Kobieta
Podziękował: 71 razy
Pomógł: 445 razy

Re: [Rozgrzewka OM][MIX][Nierówności] Nierówności

Post autor: bosa_Nike »

Ukryta treść:    
Awatar użytkownika
Premislav
Użytkownik
Użytkownik
Posty: 15687
Rejestracja: 17 sie 2012, o 13:12
Płeć: Mężczyzna
Lokalizacja: Warszawa
Podziękował: 196 razy
Pomógł: 5220 razy

Re: [Rozgrzewka OM][MIX][Nierówności] Nierówności

Post autor: Premislav »

Mnie wyszła trochę inna druga pochodna, ale i tak dokładnie taka nierówność załatwiała jej dodatniość, więc szkoda czasu na dochodzenie, kto popełnił nierzutujący na rozwiązanie drobny błąd. Możesz wrzucać następne zadanie.
bosa_Nike
Użytkownik
Użytkownik
Posty: 1664
Rejestracja: 16 cze 2006, o 15:40
Płeć: Kobieta
Podziękował: 71 razy
Pomógł: 445 razy

Re: [Rozgrzewka OM][MIX][Nierówności] Nierówności

Post autor: bosa_Nike »

Nigdy nie szkoda.:mrgreen: Rzeczywiście, powinno być \(\displaystyle{ f''(x)=\frac{\sqrt{2}}{2}\cdot\frac{\left(x^2+1\right)^{\frac{3}{2}}-\sqrt{2}x^2}{x^{\color\red{2}}\sqrt{\left(1+x^2\right)^{\color\red{3}}}}\ge\frac{\sqrt{2}}{2}\cdot\frac{\left(\frac{3}{2}-\sqrt{2}\right)x^2+1}{x^{\color\red{2}}\sqrt{\left(1+x^2\right)^{\color\red{3}}}}>0.}\) Dziękuję.


Dla liczb rzeczywistych \(\displaystyle{ a\ge b\ge 1\ge c\ge 0}\), takich że \(\displaystyle{ a+b+c=3}\)

a) udowodnij, że \(\displaystyle{ 2\le ab+bc+ca\le 3}\);
b) udowodnij, że \(\displaystyle{ a^3+b^3+c^3+\frac{45}{a^2+b^2+c^2}\le 18}\).

Wystarczy rozwiązać jeden podpunkt z powyższego zadania.
Awatar użytkownika
Premislav
Użytkownik
Użytkownik
Posty: 15687
Rejestracja: 17 sie 2012, o 13:12
Płeć: Mężczyzna
Lokalizacja: Warszawa
Podziękował: 196 razy
Pomógł: 5220 razy

Re: [Rozgrzewka OM][MIX][Nierówności] Nierówności

Post autor: Premislav »

a):    
ODPOWIEDZ